Das Ersetzen der kovarianten Ableitung U (1) U (1) U (1) durch die kovariante Ableitung GL (4, R) GL (4, R) GL (4, \ mathbb {R}) ... gibt es die Quantengravitation?

Mir ist klar, dass viele Fragen zur Ableitung der Quantengravitation in diesem Forum schon mehrfach gestellt wurden, aber nicht genau so, wie ich es hier tue. Ich würde gerne wissen, was genau ich mit dieser Ableitung bekomme; Quantengravitation, Quantenmechanik im gekrümmten Raum, etwas anderes, nichts? Wenn es Probleme damit gibt, was sind sie genau --- nicht renormierbare Transformationen von GR verletzen die Gleichungen?


Wenn ich eine Aktion definiere als

S = ψ ¯ ( ich C γ μ D μ M C 2 ) ψ 1 4 μ 0 F μ v F μ v

Dann ist dies QED.

Was ist, wenn ich die Wellenfunktion in Bezug auf eine allgemeine lineare Transformation messe:

ψ ' = G ψ G 1

Dann erhalte ich die folgende Anzeige

D μ ψ = μ ψ [ ich Q A μ , ψ ]

aber da das Messgerät im Allgemeinen linear ist, ist das Feld:

R μ v = [ D μ , D v ]

Folglich, wenn ich die folgende Aktion schreibe:

S = ψ ¯ ( ich C γ μ D μ M C 2 ) ψ 1 4 R μ v R μ v

Ist es Quantengravitation. Was sind die Probleme damit?

Warum sollte es die Schwerkraft sein? Die klassische GR ist keine Yang-Mills-Theorie, warum sollte die Quantengravitation eine sein?
@ACuriousMind Was habe ich dann bekommen ... ein Fermion, das sich in gekrümmten Raumzeiten entwickelt ... ist es zumindest das? (und eigentlich richtig bei der Beschreibung?)
Im Vergleich zu QED haben Sie nur die Gauge-Gruppe geändert und sonst nichts. Warum sollten Sie erwarten, dass eine bestimmte Wahl einer Eichgruppe eine spezielle relativistische Eichtheorie auf magische Weise in eine Theorie über gekrümmte Raumzeiten verwandelt?
@ACuriousMind Der Riemann-Tensor ist da drin ... es sei denn, er ist irgendwie gleich Null, dann tut er etwas. Ich erwarte nichts ... Ich frage, was es tut?!
@ Anon21 Die Schwerkraftwirkung ist im Riemann-Tensor linear, Ihre ist quadratisch. Ihr Lagrangian beschreibt nicht die Schwerkraft, sondern eine bestimmte Yang-Mills-Theorie in der flachen Raumzeit. Es ist auch kein Quantum, ich habe keine Ahnung, warum Sie darauf schließen würden.
@Prof.Legolasov Wie können Sie feststellen, ob es sich um Quanten oder Klassik handelt?
@ Anon21 Ich sehe keinen einzelnen Operator, der auf einen Hilbert-Raum einwirkt, also ist es definitiv kein Quantum
@Prof.Legolasov Ist der QED-Lagrangian, den ich über Quanten oder klassisch geschrieben habe?
@Anon21 es ist ein klassischer Lagrangian

Antworten (1)

Es gibt nur zwei kleine Probleme: Dieser Ansatz hat nichts mit Schwerkraft zu tun, und er ist überhaupt nicht Quanten :)

Erstens beschreibt Ihre Aktion nicht die Schwerkraft; es beschreibt die Yang-Mills-Theorie mit der Gruppe G L ( N ) = U ( 1 ) × S L ( N ) . Nicht die Schwerkraft.

Es gibt eine Formulierung der Schwerkraft in der Sprache der Eichtheorie, aber sie verwendet eine andere Aktion:

S [ e , A ] = D 4 X | det e | e A μ A B v F μ v A B
mit A A S Ö ( 3 , 1 ) Verbindung, F sein Krümmungstensor und e das Tetradenfeld, das den Tangentialraum zu einem Punkt in der Raumzeit zum abstrakten Raum abbildet R 4 und ist per Definition invertierbar.

Sie können an gewöhnliche Variablen vorbei übergeben

G μ v ( X ) = η A B e μ A ( X ) e v B ( X ) .

An Fermionen koppeln, ersetzen μ durch die kovariante Ableitung, die auf Objekte in der Spinordarstellung wirkt S Ö ( 3 , 1 ) .

Das zweite Problem ist, dass nichts davon Quanten ist. Dies ist eine völlig klassische Theorie, so klassisch sie auch sein mag. Was lässt Sie denken, dass dies eine Quantentheorie ist?

Können Sie erläutern, was Sie mit Ihrem Einwand meinen, dass es sich nicht um Quanten handelt. QED, eine bekannte Quantentheorie, ist auf die gleiche Weise definiert. Widersprechen Sie dem S = ψ ¯ ( ich C γ μ D μ M C 2 ) ψ 1 4 μ 0 F μ v F μ v ist kein Quantum, kann also nicht QED sein? Ich glaube, ich bin verwirrt von dem Kommentar.
Beschreibt mein Lagrange auch etwas Wertvolles, wie die Bewegung von Fermionen in gekrümmten Raumzeiten ... irgendetwas in der Art?
@ Anon21 QED ist ein äußerst nicht triviales Verfahren, bei dem Feynman-Diagramme aufgeschrieben, neu normalisiert und Ergebnisse als asymptotische Reihe interpretiert werden, die sich Matrixelementen von Quantenoperatoren annähert. Der Lagrange ist nur der Ausgangspunkt dieses langen, mühsamen Weges. Während es am Ende für QED funktioniert, funktioniert es bekanntermaßen nicht für die Schwerkraft, was (einer der) tatsächlichen Gründe dafür ist, dass die Quantengravitation schwierig ist. Sie haben keines der wirklichen Probleme angesprochen.
Ah ok, technisch gesehen wissen wir also noch nicht, ob mein Lagrange-Operator Quantengravitation ist oder nicht; wir werden es erst am Ende des Prozesses wissen. (Aber wir können wahrscheinlich sicher davon ausgehen, dass es fehlschlagen wird, da dies bekanntermaßen für die meisten Versuche dieser Art der Fall ist). Sind wir auf der gleichen Seite? Sorry das ist sehr verwirrend für mich.
@Anon21 keine Sorge, ich helfe gerne. Ja, das wäre größtenteils korrekt, mit Ausnahme des anderen Teils, dass Ihr Lagrangian Yang-Mills ist, nicht die Schwerkraft. (Das ist tatsächlich ein sehr wichtiger Unterschied, das sind sehr unterschiedliche Kräfte, die sich unterschiedlich verhalten, zB ist die Schwerkraft universell anziehend und Yang-Mühlen nicht).